1st series [1] [2] [3] [4] [5] [6] [7] [8] [9] [10] [11] [12] [13] [14] [15] [16] [17] [18] [19] [20] [21] [22] [23] [24]  2nd series [1] [2] [3] [4] [5] [6] [7] [8] [9] [10] [11] [12] [13] [14] [15] [16] [17] [18] [19] [20] [21] [22] [23] [24] [25] [26] [27] [28] [29] [30] [31] [32] [33] [34] [35] [36] [37] [38] [39] [40] [41] [42] [43] [44] [45] [46] [47] [48] [49]

  View the latest questions and answers at askaphilosopher.org

Ask a Philosopher: Questions and Answers 6 (1st series)

Here are some of the questions that you asked a philosopher from June — July 2000:

  1. Stoicism and Lieutenant Castillo
  2. Nietzsche and Schopenhauer on the will
  3. Are we the play things of God?
  4. The importance of Heidegger
  5. Was Hitler to blame for being Hitler?
  6. How can I learn philosophy with happiness?
  7. Why we have a right to pursue happiness
  8. What it means to 'do research'
  9. Is Being a phenomenon?
  10. Refuting psychologism
  11. What if someone falls for her mentor?
  12. Definition of 'ethnophilosophy'
  13. Plato's views on extra-terrestrial life
  14. Meaning of life
  15. Free will and evolutionary psychology
  16. Does language get in the way of self knowledge?
  17. The most important question of metaphysics
  18. Why there can't be good without evil
  19. Knowing truth just as it is
  20. Life after death
  21. Nietzsche on the eternal recurrence
  22. Reason versus meditation
  23. Existence of God
  24. Did Schopenhauer change his views on the Will?
  25. Is Hume's empiricism self-refuting?
  26. Drinking, shooting and the advancement of mankind
  27. Why philosophers are interested in etymology
  28. Definitions of 'induction' and 'deduction'
  29. Descartes on why mind can't be identical with body
  30. St Augustine and original sin
  31. Difference between mind and matter

Denis asked:

I am interested in the study of Stoicism, especially their attitudes towards the emotions. Lieutenant Castillo of the TV program Miami Vice is portrayed as the classic Stoic. My question is, Is it possible for one to accomplish such an indifference towards external circumstances? Also, do you agree with my belief that the more inner strength a person has, the more likely he or she will be able to control their emotions?

This is a great question, especially since it's so thoroughly philosophical while lying at the intersection of philosophy and psychology. Stoicism, or at least some interpretations of it, has informed Aaron Beck's Cognitive Therapy and, perhaps to an even greater extent, Albert Ellis's Rational Emotive Behavior Therapy. Both therapeutic techniques explicitly embrace the Stoic principle that our own thoughts determine our emotions. This would seem to be true in some trivial sense. If I don't learn of a loved-one's death until two days after the event then I don't feel grief until two days after she's died. Similarly, if I am misinformed that a loved-one has died whereas, in reality, she's alive and well and shopping at the mall, I will grieve for her just as though she were really dead. In some sense, then, my grief is caused by my belief that my loved-one has died and not by the actual death itself. Evaluations are even more emotively influential than beliefs, and, in fact, beliefs probably couldn't generate emotions without them. Suppose, for example, that I believe a traffic jam is going to make me late for an appointment. This belief won't lead me to feel frustrated unless I attach to it a certain evaluative judgement like "It's bad to be late for appointments in general" or "It will be awful for me if I'm late for this appointment in particular." As a first pass, then, we can say that emotions are generated and probably partially constituted by beliefs and evaluations — both cognitive things.

Of course, all of this is a far cry from saying, with the Stoics, that we can exercise control over our emotions. From our previous observations, it follows that we can control our emotions to the extent that we can control our beliefs and evaluations. Epistemology (the philosophical study of knowledge and belief) is divided over the extent to which we can determine what we believe. I think that our control over our beliefs is limited. I suppose I could retain my belief that my loved one is alive, despite being informed of her death, seeing her body, performing medical tests on her to check for signs of life and witnessing her cremation. I could believe that the body I'm seeing is actually a clone of my loved-one, for instance, but I doubt that I could willfully carry out a program of such profound self-deception. Instead, I'm willing to bet that to a large extent our beliefs are the natural outcome of the evidence, the evidence is a natural product of our senses, and our senses automatically pick up on external facts in the world. Many beliefs, then, probably come upon us from the outside (if we're physically and psychologically healthy, at any rate) so controlling our emotions by controlling our beliefs might hold only limited promise.

What's needed, probably, is control over the other cognitive element of emotion: evaluation. Judgments of good and bad seem more "internal" than judgements of fact. The Truth, as Fox Mulder says, may be out there, but the Good might not be. If my loved-one is dead, for example, that's a state of the world external to me. But the badness of my loved-one's death? Is that a state of the world external to me, too? It doesn't seem to be. Instead, that appears to be a judgement I make. If states of the world color our minds with belief from the outside in, our minds seem to color the world with evaluations from the inside out. Evaluation seems to be in our control, and since evaluation generates and constitutes emotion, if there were some way to change our evaluative patterns then we could control our emotions as well.

Which brings me (finally!) to your question. (Bet you thought I forgot it.) "Is it possible for one to accomplish such an indifference towards external circumstances?" In other words, is it possible to form correct beliefs while draining those beliefs of their evaluative significance? Since this is a matter of psychological fact, a psychologist might be more equipped to answer this part of your question than I am. Nevertheless, I'd say that it's possible. Buddhists, from what I understand, claim to have done it. More crudely, I suspect that certain kinds brain surgery could do it. Perhaps, after engaging in a program of meditation to resign myself to the impermance of all things, or after checking into the hospital to have certain parts of my cerebral cortex removed, I could accept the news of a love-one's death with complete equanimity.

Philosophically, the interesting question is should I do this? (At least that's the question that most vexes me at the moment and since I'm writing this, I can preemptorily declare this to be the philosophically most interesting question.) As you can probably tell from the tone of the previous paragraph, I don't think that we should cultivate such emotional indifferance. My reasons for this are evolving, and I might change my mind, but at the moment psychological and philosophical considerations are leading me to decry indifference. Psychologically, there are good reasons to suppose that our emotions have evolved to serve useful purposes and to believe that cultivating indifference, or even effecting a significant blunting of our emotions, in order to avoid painful or excessive feelings may be like removing a lung to cure a case of asthma. Fear helps to keep us alive. Anger helps to keep people from treating us like objects. Love helps us to care for others.

So emotions, I think, are a good thing psychologically. And I think that they're good philosophically, too. Although certain ethical theories (Kant's, for instance) hold that ethical behavior should be motivated entirely by reason, other ethical theories (like Hume's) maintain that emotions are central to moral behavior. It isn't obvious to me that Kant wins out over Hume here, and, in fact, I'm strongly inclined to bet on Hume. If I could really cultivate indifferance to the death of my loved-ones, what impact would this have on my attitudes toward and treatment of them while they are alive?

So, I guess my answer to your first question, "Is it possible for one to accomplish such an indifference towards external circumstances?" is "Yes it's possible, but the Stoics notwithstanding, it isn't necessarily desireable."

Which brings us to your second question of whether or not inner strength enables people to control their emotions. Here I think it depends upon the source of the emotion, because emotions are complicated things and come at us from a lot of different places. If the emotion, like depression, is the result of unbalanced brain chemistry, then I don't think that inner strength has much to do with controlling it. Undoubtedly, when a person is depressed, it takes extraordinary inner strength to get out of bed, to get dressed and to get to work. It takes inner strength to get to a doctor and get help, too, but I doubt that any amount of inner strength will render that help unnecessary. Sometimes people need pills, not pep talks.

There are other times, though, when inner strength is probably quite important to controlling our emotions. If I'm angry at someone, for instance, it takes inner strength to smile and behave civily. It takes even more inner strength to reframe the insult in such a way that I forgive the person and am no longer angry. Nonetheless, it can be done and sometimes, ethically, it should be done. If I feel great compassion for the suffering of others, it takes inner strength to help them. It may take even more inner strength to rid myself of the compassion. Here, however, exercising of such Herculian strength is not a virtue.

Dona Warren
Department of Philosophy
University of Wisconsin — Stevens Point

back

Wang asked:

How is Nietzsche's notion of will different from Schopenhauer's?

Both have a metaphysical definition of will in common.

Schopenhauer's notion of will is there in the title of his major work The World as Will and Representation. Everything that is, is a representation or manifestation of the will, which is universal and eternal. The world in all its manifoldness is an objectification of the will. Schopenhauer writes, "What the will wills is always life" (WWR.IV.para.54). Will is the (Kantian) thing-in-itself, the inner content (of all that is) and the essence of the world" (ibid.). Human will is a tiny mirror of universal will which is always "the will to life."

Nietzsche's notion of will as "will to power" is also metaphysical. Beware of the commentaries that treat "will to power" merely as a species of self-interest; certainly that enters into it, but it is hardly the crux. As in Schopenhauer, for Nietzsche, the words of Schelling are true: "In the final and highest instance, there is no being other than willing. Willing is primal being." (Philosophical Investigation Concerning the Nature of Human Freedom and Its Object, 1809).

Nietzsche's will to power concerns the problem of time. Nietzsche does not see time as the moving image of eternity. Starting with the notion of primal being as will, he sees time past as resisting the will, for the will is helpless before what was. "This, yes, this alone is revenge itself: the will's revulsion against time and its 'It was'" (Thus Spake Zarathustra Part II, On Deliverance).

What Nietzsche didn't like about Schopenhauer's world as will and representation was that it was world-denying. Schopenhauer thought that the world as representation and objectification was a world of illusion and instead of getting wrapped up in it, we should withdraw from it as fast as we can. If we deny our own will and discover will-lessness we will find the quietude that "alone redeems the world." (IV.65). Like Nietzsche's philosophy, Schopenhauer's demands a direct, intuitive knowledge. Unlike Schopenhauer, Nietzsche's will to power leads away from all asceticism or otherworldliness.

Nietzsche's metaphysical problem then is, how can the will redeem time? Faced with "it was", the slipping away of time, will is useless. The answer for Nietzsche is to will the eternal recurrence of the same. Zarathustra is a figure who lives the eternal recurrence and that is the 'good news' he brings mankind. Instead of repenting over the past like a Christian or turning one's back on the world as merely a realm of manifestation like Schopenhauer, Nietzsche redeems the time by willing its return and recurrence. This will to power he saw as this-world affirming. (See, The Gay Science sections, "The Greatest Stress" and "Incipit tragedia" and Thus Spake Zarathustra "On the Vision and the Riddle" and "The Stillest Hour".)

Matthew Del Nevo
www.sicetnon.com

back

Dean asked:

Could there ever be — or, one might say, is there — any external meaning to human existence? Even if God exists in the traditional sense, would we not be merely his/her play things?

How can purpose be quantified?

I can't imagine how, in general, purpose could be quantified. If a particular purpose were to achieve something or acquire something which could be quantified, e.g. to visit all the cathedrals in Britain before I die, then, I suppose, I could be said to have realised a certain percentage of that purpose.

An "after life" could be, or could contain the meaning of human existence only if it is of an entirely different order from our present experience. It would have to be such that (a) it did not give rise to the questions which generate the alleged need for a meaning in the first place (b) the answer it provided would have to be satisfactory to us; if it turned out that the purpose of this life was to fatten us up for consumption by some greedy alien in the next, or to be the toy of some frisky God, our search for meaning would not be satisfied.

The Beatific Vision, as conceived in the Christian tradition, in which all our dynamism and desires are satisfied, would be such an existence. If it turns out, on the other hand, that we are merely the playthings of God, then our aspirations would be unfulfilled and Sporty God would not be our God.

As your question implies, there may be an internal meaning to life, if there is no external meaning. There are numerous things we enjoy doing and find meaningful, even though they fill no further purpose. We do them for their own sake: talking to friends, listening to music, walking in the country, to give a few, high-minded examples. The built-in meaningfulness which we experience, if we are lucky, in certain activities in life, or sections of our lives, could, conceivably, be experienced over the whole of a life. Perhaps one might feel this if one had a strong sense of mission or a highly developed aesthetic sense of the unity of life. Whatever generates this sense, it would have to be something strong enough to overcome the depressing thought that life is just one damned thing after another.

Even if we come to believe that life does not need a purpose external to itself to be meaningful, death may remain a matter of regret. The meaninglessness of life and the awfulness of death are two ideas that have often been run together, but they should be uncoupled. Finding life meaningful does not necessarily reconcile me to its termination. There may be, as Thomas Nagel suggests in Mortal Questions, a bad end in store for us all.

Michael Bavidge
Deputy Director
Centre for Lifelong Learning
University of Newcastle

back

Andrea asked:

I would like to know why a philosopher like Heidegger motivated such controversial opinions like the one of Carnap and others. Which are the most important advances in his thought?

The statement by Heidegger most frequently paraded around by logical positivists such as Carnap and Ayer, as the supreme example of metaphysical nonsense, is 'Nothing Nuths' Das Nichts Nichtet, taken from his Introduction to Metaphysics. I doubt whether any of them bothered themselves to actually read the work.

David Farrell Krell gives a judicious summing up of Heidegger's contribution to philosophy in his article for the Routledge Concise Encyclopaedia of Western Philosophy and Philosophers edited by Urmson and Ree, 2nd Edn 1975:

Heidegger is without doubt the most powerfully original and influential philosopher of the century in the Continental tradition. Phenomenology, existentialism and deconstruction are unthinkable without him, but so are philosophy of literature and many social-critical or neo-Marxian strands of thought. His importance in the English-speaking world is also growing steadily, because no other contemporary thinker so unsettles the analytic enterprise. While convinced analytical philosophers fulminate against him, or try to ignore him, their students have long been reading him.

For me, the most important advance in Heidegger's thought was his discovery of a new way of doing metaphysics. Being and Time, published in 1927, ranks alongside the greatest works of twentieth century metaphysics. It may not be easy going, but it is still a lot more readable than, say, Whitehead's Process and Reality or McTaggart's Nature of Existence. In the phenomenological tradition, Emmanuel Levinas Totality and Infinity is the only work of equal stature.

While Whitehead and McTaggart follow the traditional route of constructing a 'theory' of existence, delineating the true structure of reality hidden behind, or only dimly perceived through the distorting screen of ordinary language, Heidegger sees our actual everyday existence in the world, rather than merely the things we say, or the concepts we use to categorize the things around us, as implicitly involving a certain relation to reality, or 'Being'.

In Heidegger's picture, you and I do not merely hold a false view of reality, as Whitehead or McTaggart would claim. Rather, we live a false, 'inauthentic' existence. This is not moralizing, nor is it theorizing. It would be more correct to say that what Heidegger is looking for is a change of attitude that would halt, and then reverse the process of the ever-increasing concealment of 'Being'. Being is at least partially revealed when we rediscover the question of our own existence, in facing up to the absolute inevitability of our own death. Therein lies the clue to the metaphysics of time. Just where one goes from there, however, is not entirely clear. One gains the sense that Heidegger remains absorbed with clearing away the obstacles in the way of Being, without a thought to what we would actually see when the last obstacle is removed.

Geoffrey Klempner

back

Nick asked:

Can we hold a criminal responsible for his actions when he was either born predisposed to these actions or conditioned to do so? When I ask religious authorities this question, a third element is often added — something called 'free will'. This is never given much explanation and therefore I do not see how this fits in. I'd appreciate any help on this topic.

They say that to understand is to forgive. If we understood Hitler, could we forgive him? Imagine that you are the recording angel, watching ever stage of Hitler's life from the moment of conception, through birth, childhood, adolescence to adulthood. The formation of Hitler's character, the decisions he made or the events that influenced him become transparent to the angel's gaze. Each step along the way is either fully intelligible in the light of what went before, or else a result of mere chance, a roll of the dice.

From the angel's perspective, watching Hitler's life is like watching the formation and process of a natural disaster. There is no point in the attitude of moral condemnation, any more than there would be in shaking one's fist at a forest fire, or putting an earthquake on trial.

I don't think that it is enough to say that we don't occupy the angel's perspective, and leave it at that. If we do not see what the angel sees, then surely that is a matter of our ignorance.

Much is usually made at this point about the necessary role of interpersonal attitudes, where individuals are 'held responsible' for what they do, or, more crudely, the necessary function served by punishment or making an example. It doesn't answer your question. You can see through all that. You will say that the trial and execution of Hitler would be a charade, just as every trial and sentence is ultimately a charade. The thing philosophers call the 'moral world' is a reality constituted by its own mere appearance, a world of phantasms that we mistake for substantial things.

I am not ready to make the step of renouncing the moral world, because I am afraid of where it would lead. Call it a fondness for comforting illusions if you like. Ridding ourselves of these so-called illusions would be the first step towards a descent to Hell and the ultimate extinction of humanity. In place of one Hitler, we would have a thousand.

Geoffrey Klempner

back

Rocio asked,

I have long been interested in the subject of animal rights and the human relation to them regarding ethics. Kant would probably start the discussion with a reference to our superior rational nature. However, isn't this being a bit anthropocentric? If rationality and intelligence capacity is the factor for value then we would be justified in treating mentally retarded children like we do cows or chickens. There must be a basis for the assumption that humans have a greater value besides just, 'Well, we're human.'

Many would argue that we have the capacity for moral reasoning and animals do not, but this seems more of an argument for the moral burden of caring for sentient life that does not have our capacities. A lack of moral reasoning in a retarded child would oblige us to care for them to a greater extent than a person with full capacities.

So with this lengthy preface, What is the best way to account for the human value system, given the ethical questions involved? In other words, how could we justify valuing ourselves above other sentient life forms?

What does it mean: to value ourselves above other sentient life forms? If it means that we always put our own interests above the interests of other species, there is no justification. It sounds like selfishness, pure and simple. Being rational, and therefore highly imaginative and empathetic, we care about lots of things other than ourselves. We would not ourselves be so wonderful if we did not have the capacity to care about other people, other species, mountains, music and the Andromeda Galaxy. Our values are our values, but that does not mean that the only things we value are our own interests.

If animals are incapable of moral reasoning then the sort of moral attention centred on others precisely as moral reasoners, does not apply. Some people argue, for example, that only moral reasoners can have rights. But even if that were true, it would not exhaust our moral concern. The Body Shop, in promoting its animal friendly policy, used Bentham's famous remark "The question is not, can they reason? nor, Can they talk? but can they suffer?". Animals experience pain and they have interests. Most of us believe that we ought to consider their welfare as well as our own.

However, an animal's experience of pain is affected by its emotional and imaginative resources. The human animal experiences despair, horror and outrage at injustice. In my opinion, non-human animals do not experience this depth of spiritual suffering, though many are capable of depression and anxiety.

In discussing issues of animal welfare, we ought to be very careful in drawing comparisons — as you do in your question — with hard human cases. The fact that a damaged human being is (a bit!!) like a healthy non-human animal raises some questions, but it does not resolve them. A bird with a broken wing is not very like a dog because it cannot fly. It is a completely open question whether two quite different creatures ought, on the basis of some similarities in capacity, to be treated in similar ways. My instinct, like yours, is that people with special needs have a special claim to care.

There is another sense in which we could be said to value ourselves above other sentient life forms: we may place rational capacities above physical capacities in a hierarchy of value. How would you rate being good at theoretical physics (Einstein) in relation to looking pretty (a bird of Paradise)? This exercise sounds like fun, but, as a thought experiment designed to test our scale of values, it should be treated with caution: what if the house is on fire and you have to choose between saving granny, the baby, the last surviving manuscript of Beethoven's Fifth or the budgie? Part of being so darned clever is the ability to avoid having to make these choices. Buy a smoke alarm. But what if the alarm fails?...

Ok if I have to play, I would save the baby, (invest in the future), wring the budgie's neck (so that it has a quick death), forget about the manuscript (having faith in the future creative powers of the human race) and kiss granny goodbye (I wouldn't kill her because, you never know, something may turn up, so she survives against all the odds).

There are, of course, non-flippant issues of prioritising in, for example, the provision of medical services. And sometimes choices can become tragic or desperate. During World War II, parents in the Jewish ghetto at Vilna had to choose either to hand their children over to be killed or to refuse and have the Nazi soldiers come in and take them by force. Such moral traumas create upheavals in our moral world. They are Nietzsche's "ploughshare of evil". These transformation in the moral landscape go too deep in the ourselves and our society to be captured in tick-list of priorities.

Incidentally, Kant's concern with rationality, he would say, is not anthropocentric, because what attracts Kantian moral respect is not the human, but the rational element in human reason. Any other form of rational existence, if any, would have the same moral value.

Michael Bavidge
Deputy Director
Centre for Lifelong Learning
University of Newcastle

back

Kanokwan asked:

Maybe I must take the course for Philosophy in the University...so I need some suggestions.

How can I learn Philosophy with happiness and make the point so very good?

The first piece of advice I would give is to look very carefully at the Department of Philosophy at the University where you hope to study before accepting the offer of a place. Pay them a visit, or, better still, several visits. Who is going to teach you? Are the lecturers and professors enthusiasts, or do they regard teaching as a necessary chore, and students as a mere inconvenience?

How can you learn philosophy with happiness? So few students even think to ask that question. Their only concern seems to be with success and doing well. Would you still be prepared to carry on even if you did badly, and found yourself at the bottom of the class? The rewards of studying philosophy are not reserved for the high flyers. However, in order to reap those rewards you need to adopt an attitude of humility. In the face of the perennial problems of philosophy we are all ignorant and stupid. Some are slightly less so, that is all.

Take your time, don't bite off more than you can chew. There is no point in toiling away for some possibly future reward, if you can't enjoy what you are doing here and now, in the present. The great moments of illumination will come when they come. But meanwhile every moment contains the possibility for enlightenment if you look for it. Enjoy the journey, because in philosophy you really can't say whether you will ever reach a destination.

Read about the lives of the great philosophers, and gain inspiration from them. Read, not to idolize, nor simply to appropriate ideas for your own use, but to bring the words on the page to life. Every text you pick up is an invitation to dialogue. The greatest joy in reading the classics of philosophy is the feeling of entering into a dialogue that transcends time and space. The great philosophers live on in their works, and that is where we can meet them whenever we choose.

Understandably, you also want to 'make the point so very good' — you want to get good marks for your essays, make a big impression in seminars and discussions. To do that, you need to overcome the fear of failure. Sometimes, the only way to learn is from our mistakes. You must not be afraid to look a fool. That means not getting sucked into to a way of thinking where everyone is in competition with you, where you either win or lose. Let others compete if they want to. Show that your only concern is understanding and learning more, that for you truth is more important than being perceived to be right. That will make the best impression of all.

Geoffrey Klempner

back

Aimee asked:

In the context of the American Declaration of Independence, the concept of unalienable rights is of course understandable if God is accepted. However, the phrase, 'pursuit of happiness' is giving me a problem. surely, this is not a declaration of Hedonism, but what exactly does this phrase claim? Is it better to say, a right to self-fulfilment or a right or opportunity to resources to make someone happy?

It says in the American Declaration of Independence that,

We hold these truths to be self-evident, that all men are created equal, that they are endowed by their Creator with certain unalienable Rights, that among these are Life, Liberty, and the pursuit of Happiness. That to secure these rights, Governments are instituted among Men, deriving their just powers from the consent of the governed...

Just what does that troublesome phrase, 'the pursuit of happiness' mean?

It is not a declaration of Hedonism. To say that one has a right to the pursuit of happiness does not imply the — in my view, quite implausible — theory that every human action is ultimately motivated by the desire for one's own pleasure or happiness, still less that every action is regarded by the agent as a mere means towards the end of enjoying some pleasure, or feeling happy or contented.

I agree with you that the idea that life and liberty are not enough, that something needs to be added to account for our God-given rights, has something to do with the availability of resources. What use is liberty, the absence of interference or constraint in setting out the plan of your own life, if the lack of resources means that there is no possibility that your plan could ever be brought to fruition? Everyone has the right to a decent chance, that in plain terms is what the Declaration of Independence is saying. We intuitively feel this to be true. A government that allowed a sector of the population to be deprived of a decent chance, would be failing in its duty to its citizens.

Equally, the right to the necessary resources would not make up for the lack of liberty. Suppose a government said, 'We will give you all the resources, all the opportunities for what we deem to be your 'self-fulfilment'. That would be an apt description of the illiberal state described in Plato's Republic. It clearly does not satisfy the conditions laid down by the Declaration of Independence. Before the iron curtain fell, Communist regimes boasted that their citizens' 'economic freedom' more than made up for their lack of political freedom. — But they were wrong.

Geoffrey Klempner

back

Maria asked:

I have several questions: Is there really a philosophy behind the action of doing research? and if so what kind of a philosophy would it be? Is research a solitary attitude, a need or a conviction? Can research actually be shared with others, thus is interdisciplinary research really possible or is meta-science only a myth?

Certainly, there can be philosophy about the action of doing research. Two books in particular come to mind:

Feyerabend, in Against Method, attacks the notion of a 'scientific method' that can be stated in general principles or rules, a method which if we follow it with care will lead us reliably to our goal. Practical, real-life science throws rules and principles out the window. Habermas, in Knowledge and Human Interests argues against the 'disinterested' view of the researcher, which views human knowledge in abstraction from its social context, and the uses to which knowledge is put. In both cases, what is being radically questioned is Descartes' view of human knowledge as essentially the achievement of a solitary subject, oblivious to everything except its own 'clear and distinct ideas'.

Philosophers in the analytic tradition — for example Michael Dummett in his seminal 1973 book Frege: Philosophy of Language — have laid heavy stress on the way Wittgenstein's later work displaced the theory of knowledge from the central place that it had occupied since Descartes. But it is fair to say that the philosophy of science has been completely ignored in this over-simplified picture. If I have to view myself as an agent in the world, rather than standing outside it, using language, concepts which I share with others and which I could not have invented for myself, then certain consequences for 'the activity of doing research' follow.

Philosophers of science still seem to be obsessed with the question of what is 'good' science or what is 'bad' science, as if the scientific community were waiting for their pearls of advice, the stamp of approval for their activities. More than anything, the philosophical injunction to 'know ourselves' requires that we accept science and research as political and sociological phenomena. We play games. Language games, certainly, but also the historical research game, the literary research game, the chemistry research game. — The philosophy research game.

I had a teacher once at school who was working on his PhD thesis in Physical Chemistry. He was trying to determine the melting point of a delicate crystalline substance, which required elaborate vacuum apparatus and miles of glass tubing. It was his brick that he would one day lay, with great ceremony, on the ever-rising edifice of science. It was also an exercise, to show that he knew how to 'do research'. Apart from occasional meetings with his thesis supervisor, however, he worked alone in the odd hours in between classes. Was he doing research, as he thought? In a way, he wasn't.

Well, maybe you will disagree. If my teacher ever did succeed in determining the melting point of the crystalline substance (I shall never know whether he did or not) then that was a potentially useful result. But there was also something that he was not learning or practising: how to collaborate, how to be part of a research team.

Potentially, all research is inter-disciplinary research. What I mean by that is that research, viewed as a naturally collaborative enterprise, inevitably involves some form of division of labour. The various members of the research team are not necessarily inter-changeable, they could not all do one another's jobs. In that case, you will ask, how can they justify their actions in the face of criticisms from their colleagues. How can I, the non-expert, pass judgements on you, the expert? — If you think about it, the response to this is straight-forward. As a matter of fact, we do, all the time. That is what you are doing now.

Geoffrey Klempner

back

J. Holder asked:

Is Being a phenomenon?

Possibly.

The question of the Being of the universe is not a question about something that lies behind the phenomena. There is no world of Kantian 'things in themselves', forever beyond the reach of human knowledge, known only to the Diety. Yet nor is the Being of the universe a meaningless question.

Haven't I just exhausted the alternatives?

I would argue that there may, possibly be a third alternative. I say 'possibly' because the jury is still out. The question of the 'Being' of the universe concerns things, or 'beings' as a whole, as a totality. The fundamental question of metaphysics is whether there could possibly be anything meaningful to say about the universe as a whole, or as Aristotle put it, about 'Being qua Being'.

If Being is a phenomenon, then there is something that metaphysical inquiry brings us to see. Being can be made to 'appear'. Here is what I say in unit 1 of the Pathways metaphysics program The Ultimate Nature of Things:

Metaphysics is essentially transcendent. In attempting to break free from our immersion in mundane things, however, as metaphysicians have traditionally sought to do, the contemporary metaphysician no longer seeks a world of absolutes — purer, fairer, more 'necessary' than the transient world of phenomena — but merely to increase the angle of view on our own familiar world. 'Defining reality' in this relatively modest sense means stepping outside the mundane world (just one step would be enough) in order to grasp it synoptically as a world, a reality, something that mind, or the 'I' stands in relation to.

Of course, this is a very abstract account of the problem. The only way I know how to make it more concrete would be to tell you my theory. And I don't want my answer to your question about Being to be contingent on the truth of my theory. (For an overview, see Naive Metaphysics, Chapter One.)

In other words, I think it is more likely that my theory is wrong, than that I am wrong about the possibility of 'stepping outside the mundane world' in order to gain a view of the phenomenon of Being.

Geoffrey Klempner

back

Joan asked:

Does Husserl succeed in refuting psychologism?

What is psychologism, and why should anyone be bothered to refute it?

Edmund Husserl was concerned with psychologism in the account of logic, and logical inference. If you ask an ordinary person what 'logic' is, they will reply that it is something to do with the way we think. To think logically is to think rationally, to think illogically is to think irrationally.

So what are the laws of logic? A person thinks irrationally when their power of reasoning is impaired in some way. For example, people think and act irrationally under the influence of strong emotions, or strong liquor. It seems a natural inference to draw that the laws of logic describe the way people think when their thinking is not impaired.

If that is true, then it follows that the laws of logic are very general laws of psychology. In a similar way, opthalmology is concerned with the way people see when their sight is not impaired. There are tests that determine whether or not a person is colour blind, or whether they are long sighted or short sighted. 20-20 vision is the norm. The fact that our eyes cannot detect ultra-violet radiation, or that we lack the acuity of vision of an eagle, is not an impairment. It is just a natural fact about the way human beings are constructed.

What I have just described is psychologism about logic. In his book Logical Investigations Husserl gave a powerful argument against psychologism, which according to many philosophers laid the ghost of psychologism to rest forever. In order to grasp the point of his argument, we need to take things slowly. Consider the following statement:

You can't have your cake and eat it.

Say you have a delicious portion of Sacher torte. You can either eat it now, or you can keep it for later. I can spend all my money on a holiday, or I can save it up for a rainy day. This is a kind of choice we face many times in life. Parents are all-too aware that young children can't always see this. They want it both ways. The situation has to be patiently explained to them.

Now the saying, 'You can't have your cake and eat it,' is one familiar consequence of the logical law of non-contradiction:

It is not the case that (P and not-P)

Husserl argued that the scope of a logical law, like the one I have just quoted, is unrestricted in a way that could not be explained if the laws of logic were mere laws of psychology. The law applies to all objects, all places, all times. If the law of non-contradiction were merely a psychological law describing the way human beings think when their thinking is unimpaired, then it would be perfectly possible that we might one day encounter aliens (from the planet Hegel) whose thinking, when unimpaired according to their norms, did not obey the law of non-contradiction. Instead, on the planet Hegel, it is a law of logic that:

It is not the case that (P or not-P)

'Isn't that absurd!' you might think. Well, I'm not entirely convinced by that argument. The problem is, that every attempt to justify our belief that the so-called laws of logic are valid for all objects, all places, all times turns out to be blatantly circular. It would never convince a determined sceptic. The bottom line is that we cannot conceive of what it would be to possess the mind of an alien from the planet Hegel. And the reason is that conceiving is thinking, and we can only think the way we think. Round and round you go.

In other words, the situation is exactly as it would be if the laws of logic were merely psychological laws describing the way human beings feel impelled to think.

I don't think it is fair to say that Husserl failed to refute psychologism. He succeeded (with some prompting, some would argue, from the mathematician Gottlob Frege) in the lesser task of laying the foundations for an autonomous discipline of logic, uncontaminated by psychology. And for that he is rightly remembered.

Geoffrey Klempner

back

Dian asked:

What if someone falls for her mentor? Could Platonic eros be "true" love? What do the "roots" of Empedocles, the Platonic fluids and the Wiccan elements have in common? why is Plotinus described as the last great pagan philosopher? Generally, how is Ancient Greek philosophy related to magick?

What are the principles of religious atheism? If one is too respectful of the world to be an atheist but also too faithless to be a theist or a pantheist, where does one stand?

Are emotions "real"? I'm using myself; meaning that I choose to become depressed and I end up crying, I then command myself to feel enthusiastic or irritated and it happens until I get bored enough to give a new order. Where is my consciousness when I do this? Who is commanding and who's obeying? I can't feel unless I choose to. So is our world an acting stage? Who are we behind the masks we choose to wear? Is there a face at all behind these masks or are we just a big nothing?

Is the soul a part of our brain? Wouldn't that mean when our brain stops functioning,the soul gets lost? Then what in case of severe brain damage?

Why can't I stop the flow of thoughts until I drop from tiredness? Could you quit philosophy? It seems, one can't claim he likes philosophizing; maybe there's something seriously wrong with his mind and he can't do otherwise! Do we find it great because we can't help it?

You should never fall for your mentor. It's always a mistake. The other way round, though, is quite acceptable. Socrates was a better teacher for having fallen for Alcibiades. When you love someone, you have all the more reason to nurture and care for their soul. But Alcibiades was a worse pupil for having fallen for Socrates, as his subsequent behaviour seems to have borne out. — For those who are wondering what I'm talking about, it's all in Plato's dialogue Symposium.

What are the principles of religious atheism? There are various positions you can try to defend. You can say that what you mean by 'God' is simply The Good. In other words, you believe, without being able to offer any further justification for that belief, that there is an objective difference between 'right' and 'wrong'. Not everyone has sufficient faith for that. (For some, doing the right thing is simply the easiest option, it gets us into the least trouble in the long run.) Or you can say that you have a sense of awe at the existence of the world. You experience something that might be called the religious attitude in the absence of any religious belief. Not every would understand what you were talking about. There is a third option:

Respect for other persons is the recognition of their essential otherness, the fact that they occupy a standpoint that I can never attain, a standpoint that gives them the authority to evaluate my judgements. In this sense, all other persons are equal in my eyes, and may indeed be thought of as ultimately one: the Other. If I permit myself to harm this person, say, the leering tramp waving his angry fist at me, if I fail to respect his otherness, then I have in effect committed a crime against the Other as such, a crime against 'the Deity'. We have argued that it is only through my recognition of the Other that there comes to be an objective world for me. In that metaphorical or mythical sense, therefore, that philosophy vindicates the religious view of the world and all the things in it as 'God's creation'.
Pathways Program B. Searching for the Soul Unit 15.

The difficult part is knowing what you have to do. I mean, it is hard enough deciding what is best for our own selves. If a junkie asks me for money, which I know is going to be used on drugs, do I give them what they want, or refuse, and give them my packed lunch instead? Which is the moral thing to do? Which is more respectful of their 'otherness'?

I'll skip the other questions this time, if you don't mind!

Geoffrey Klempner

back

Mary asked:

Could you give me a brief definition of "ethnophilosophy"?

If you looked at this page earlier, I apologize for having originally rephrased your question — something I don't usually do — as, 'What is "ethnophilosophy"?' I thought it would be too easy just to give a brief definition. I couldn't have been more wrong!

Ethnophilosophy studies the philosophical beliefs and world view of 'the Maori', 'the Azande', 'the Hopi'. I have deliberately chosen peoples from three different Continents, because although the original idea of ethnophilosophy arose within an African context, it would clearly be wrong to define it in such a way that it was impossible to do ethnophilosophy outside of Africa.

The first thing to note is that we are not talking about any individual Maoris, or members of the Azande tribe, or Hopi Indians. The ethnophilosophy of a people does not have a history, in the way that one might speak of the history of Eastern or Western philosophy, or of the Continental or Analytic traditions. Whatever developments may take within an ethnophilosophy are not the result of successive contributions to an on-going debate, but rather follow the vicissitudes in the history of the society which has given rise to it.

Some academic philosophers would argue that what that shows is that ethnophilosophy cannot be 'philosophy' in any recognizable sense. The very idea involves a contradiction. - But then those philosophers first owe us a non-question begging definition of 'philosophy'.

The issue over ethnophilosophy carries strange echoes of an old academic controversy surrounding the beginnings of Greek philosophy, the logos over mythos debate. In one account of the history of Greek thought, the achievements of the Presocratic philosophers represented the 'triumph of logos over mythos'. In the place of uncritically accepted creation myths, there arose the idea of rational inquiry as the source of theories about the universe. According to the rival account, reason did not suddenly appear on the scene, as if from nowhere. There was a gradual development, with mythical elements continuing to play a significant role in the theories of the first 'philosophers'.

My view? Philosophy is about reasoning and argumentation. Philosophy is also about the different ways in which the world can make sense for us. It can be one, without being the other. So I have no difficulty with the thought that ethnophilosophy is philosophy properly so-called.

Geoffrey Klempner

back

Casibjorn asked:

They say that Plato said everything in Philosophy and everything that follows is but a reflection of what he has written. Currently I'm a student in the Philippines and a seminarian, and I am questioning about the existence of extra-terrestrial life. In cosmology as well as in metaphysics this question is raised but I need a second opinion, especially from a true philosopher not just a master.

Hoping you will answer, God bless and peace.

It was the metaphysician Alfred North Whitehead who said:

The safest general characterization of the European philosophical tradition is that it consists of a series of footnotes to Plato. I do not mean the systematic scheme of thought which scholars have doubtfully extracted from his writings. I allude to the wealth of general ideas scattered through them...
Process and Reality Corrected Edn Macmillan 1978 p. 39

Here is Plato's view on extra-terrestrial life:

Are we right in saying that there is one world, or that they are many and infinite? There must be one only, if the created copy is to accord with the original. For that which includes all other intelligible creatures cannot have a second or companion; in that case there would be need of another living being which would include both, and of which they would be parts, and the likeness would be more truly said to resemble not them, but that other which included them. In order then that the world might be solitary, like the perfect animal, the creator made not two worlds or an infinite number of them; but there is and ever will be one only-begotten and created heaven.
Plato Timaeus translated by Benjamin Jowett

I wonder whether it is this passage that has inspired your question?

Well, is there life on Mars? Or if not on Mars, somewhere in the wide universe? It is difficult to conceive how there can be a logical reason why there cannot be other worlds inhabited with intelligent life, even on the assumption of a Creator. If an artist paints the perfect painting, then perhaps he has no need to ever paint again. But we cannot reliably second-guess God's purposes.

The alternative is to go for the best theory. At the present time, there is a massive project underway to try to detect radio transmissions that might have emanated from extra-terrestrial life. The lack of positive results may only be an indication that intelligent life is not very common in the universe. So the question comes down to probabilities. Assume such-and-such as the probability that intelligent life evolves on Earth. Estimate the total number of planets in the universe where life had the chance to evolve, do a simple calculation and out comes the answer.

I am naturally curious to know whether there are others, somewhere, like us. Until we actually meet up with creatures from another planet, however, the answer is not a foregone conclusion. It is still possible that the initial probability of intelligent life evolving on an Earth-like planet, is much, much lower than the current estimate. In other words, it is possible that we are, indeed alone.

Geoffrey Klempner

back

Altug asked:

What's the meaning of life?

There are short answers to this question, like 'love', 'happiness', 'to live truly', 'to truly live', 'life has no meaning beyond that which you give it' etc. If you can find a short answer that seems to answer your question, that is a good place to begin further reflection, if further reflection is needed, which depends how strongly you are wondering.

Then there are metaphysical answers to the question. These are the answers most favoured by philosophers and therefore thought most philosophical. Metaphysical answers are characterised by their embracing quality or comprehensiveness. Religions provide meaningfulness in the lives of many millions. Religions provide metaphysical answers, for instance about the nature of God (e.g. as Creator), the acts of God (e.g. giving us a Law or becoming a human himself) and our relationship with Him (e.g. He loves us, He has given us the sovereignty of free will and so He cannot have power in the world without except through mankind). Metaphysical answers in religion tend to blend with mythological answers, but modern Western philosophers tend not to subscribe to myth.

Philosophers have metaphysical answers of their own, for instance, that Philosophy herself is the meaning of life, or if it isn't, it certainly gives meaning to life. Another example: the meaning of life lies with the ever greater understanding of it. Or the meaning of life is being worked out in (or by) history. Or the meaning of life is hidden in the soul (hence psychotherapy).

All these ways of answering tend, perhaps, to make the questioner feel that the meaning of life is uncertain, and, therefore, that those who say there isn't any meaning, could be right in their judgement. Although how do we know what the criteria for the judgement of this question are?

I would venture that the meaning of life is different for different people. Therefore the meaning of life is not this or that but it depends who you are! The first question on the way then is: Who Am I? This question is not just an abstract question, but it leads to the discovery of meaning in life. This is not necessarily to say that the meaning of life is 'relative'. One of the best places to start to find an answer to the question of who I am, which directs me forcefully toward the meaning of life is the thought of death. I like Nietzsche's aphorism in the Gay Science, entitled, The Thought of Death. Or best of all, Tolstoy's novel, The Death of Ivan Ilyich. Confronting the question of death, through reading and imagining, brings us into a sense of the meaning of life. We often imagine dying is something that 'happens', like an event, at the end of life, but being mortal, such as we all are, means I am dying now — even I am as I write this and you as you read it. When you can think of death (and it is only modern Western culture that in the history of the world has avoided this thought) you know what is important and what it not; you know what is meaningful and what is not. Not because you know something more than you knew before, not because you are better informed by philosophy, but because you can SEE now what before you could not. The meaning of life is not a datum, it is something that must dawn upon us. The question, Who am I? posed in tandem with the thought of death clears our vision so that the meaning of life can be beheld.

Matthew Del Nevo
www.sicetnon.com

back

Chris asked:

Does evolutionary psychology imply we have no free will?

Let's not beat about the bush. When people talk of 'evolutionary psychology' they are usually thinking of sex.

Here's an illustrative example. Much fuss was made a while ago of research which claimed that there were sound evolutionary reasons for female infidelity. Up until then, the received view had been that according to the 'selfish gene' principle, the human male had the best chance of propagating his genes if he had intercourse with as many females as possible, while the human female had the greatest stake in maintaining a monogamous relationship, in order to give her children the best chance for survival. The new research appeared to demonstrate that female monogamy is ideally combined with opportunistic couplings with males who are perceived to possess superior genes.

Some people found this shocking. The fact that they were so shocked says a great deal about the continuing hypocrisy of present day sexual attitudes.

The more thoughtful and less sexually biased observer might still find reason for disquietude. We think of a decision to be unfaithful as a pre-eminent example of the exercise of free will. It turns out that the unfaithful wife is responding blindly to the commands of her genes.

Now, it is always open to an opponent of free will to adopt the trumping tactic of claiming that every deliberate human action, insofar as it is the end product of a chain of causes and effects, cannot be described as truly 'free'. There are powerful counter moves against that argument, however. What we are now concerned with is an additional reason for concern, based not on metaphysical dogma, but on empirical research.

I am not convinced!

What are we really saying? Human beings have a nature. We inherit natural predispositions from our genes. That is hardly surprising. It would, if anything, be a far greater cause for concern if it turned out that human psychology is infinitely malleable. That there was no such thing, from the inside, as what it is to be human. Then it would be completely up to us to make of ourselves what we will. What a terrible burden that would impose!

It is truistic that it does not follow from the fact that human beings have a nature, that everything that is 'natural' to us is desirable. Every responsible human parent — as I can testify from personal experience — faces a battle against biological nature. (It was left to Freud to give this homely observation a vicious twist in his late work Civilization and its Discontents.)

In short, to be in possession of a capacity for reason is to be capable of making choices. If we submit to 'nature' then we are acting for a reason, which we may reflect upon and which others may praise or criticize, no less than if we resist.

Geoffrey Klempner

back

Anthony asked:

To what extent does language obfuscate our vision of ourselves? While it is of undoubted relevance to our interactions with other people, I feel that people rely on it to 'converse' with themselves. Why is it needed when we think to ourselves, given that the act of forming a feeling into words will always lose something in translation. I don't believe it is necessary. Relying on language to explain our own feelings to ourselves will ultimately distance us from our true selves. Often, the first thing we feel is the right thing eventually. Do we not trust our instincts? is this concentration on language a result of modern man's immersion in the physical world?

The question of language is probably the dominant philosophical question of the twentieth century. Heidegger said, "all thinking is experience with language" (On the Nature of Language). There is no thinking about language outside it. Language can never be something merely at hand, like a tool, because we are always in it, like an element or a universe. Objectivity is a stance taken up within the sphere of language, which uses language after a particular prefabricated fashion. In Heidegger's words, one has always already been "claimed" by language, by the very questions we ask, which reflect our experience of language (as far as it goes). Our relation to language is the same as our relation to all that is. "No matter how we put our questions to language about its nature, first of all it is needful that language vouchsafe itself to us. If it does, the nature of language becomes the grant of its essential being, that is, the being of language become the language of being" (ibid.).

The idea here is a bit like that of the later Wittgenstein, who came to see that "the meaning of a word is its use in language," (Philosophical Investigations) and that the use of language belongs to whole "forms of life". For instance, scientists derive the meaning of their words from the way that they are used within their professional world. Moreover, this world bespeaks a stance to beings as a whole, that of the methodologist. 'Things' therefore show up in a scientific light. The same things may be quite otherwise for the poet. Our stance in language is language's "claim" upon us. Language grants itself to us in different ways according to our stance.

The great danger with questions of language, and I fear your questions reflect this, is the fallacy of subjectivism on one hand and logic (in the positivistic sense) on the other. Language is not words which form a 'picture' of the world, or which stand for 'things' in that world or in oneself. Language is not a matter of the first person singular (the ego). Then language falls out of its own element (which is the Being of beings according to Heidegger or forms of life according to Wittgenstein) and becomes conceived in terms of psychology and subjectivism. Language remains a matter in question and any straightforward answers, especially of an analytical nature, would be preemptive.

Matthew Del Nevo
www.sicetnon.com

back

John asked:

What do you consider to be the most important questions of metaphysics?

According the Heidegger in his Introduction to Metaphysics (1953) the first question is: "Why are there things that are, rather than nothing?" Heidegger means 'first' in a metaphysical sense not a chronological sense; he thinks that this is the most underlying of all questions. This is not just his idea, it comes from (his interpretation of) Aristotle. All our answers to every other philosophical question presuppose the way in which we have resolved the Seinsfrage (question of the Being of beings). The question of being is the most important question of metaphysics.

Matthew Del Nevo
www.sicetnon.com

back

Noah asked:

Without evil, how would we know what "good" was? Don't we need "evil" to achieve balance, just as we need winter to appreciate the summer?

In the Nichomachean Ethics — a document which is formative for Western culture — Aristotle noticed that the desire for happiness was universal. It didn't matter who you were, what your background was or what you did for a living, everyone seeks to be happy. Happiness is the good. He went on to observe that false consciousness is almost a norm. This is when people seek happiness by ways and means that will only lead in the opposite direction, towards unhappiness and even self-destruction. He distinguished therefore between two types of good: the true good which will lead to happiness and the false good, which glitters like the real thing, but which will lead us away from the happiness we seek. Aristotle argued that happiness was to be sought for in oneself. Those who have found peace of heart are happy. To find peace of heart there are the practical arrangements of our existence which need taking care of first of all. Expensive food, fine houses, servants and possessions will not make us happy in and of themselves. Happiness is a matter of the heart. Aristotle recommended moderation with respect to our physical lot in life, else it distract us from the happiness we desire. Once we have sufficient to meet our physical needs then the real search begins. Aristotle argued that those who are happiest in themselves are the virtuous. Virtue is an aid to happiness. Our good lies in our virtue.

The balance, Aristotle taught, was between different ways of finding the good. Good government required balancing conflicting individual goods to find the common good.

You can see that we do not need 'evil' to know what good is. We know good naturally, we know ill by default, Aristotle thought.

It was Christianity which moralised and polarised the concept of the good. Suddenly the good was equated with God (and His earthly representatives) and evil with Godlessness or lack of faith. Classic Christianity taught that evil is a privation (absence) of God's holy presence in our lives and in our world. This presence alone is good. 'Evil' is not native to philosophy, it is a religious judgement. To the extent to which, philosophically, the world is Christianised, we can hardly do without evil, as it were. Yet evil is a thorn in the side of Christianism as well, since the question as to why a loving all-knowing and all-powerful God allows evil to exist is unanswerable. Either God is not all-loving or He is not all-knowing and all-powerful. Yet neither of these answers are acceptable to Christian religion. To answer your question directly: good and evil are not a yin-yang thing. Good is native to the human spirit, evil arises when virtue is wanting. Evil is good gone wrong, sometimes very wrong.

Matthew Del Nevo
www.sicetnon.com

back

George asked:

If the truth is indeed 'out-there', where is the location of it? To put it another way: if simulation or mediation are to be considered inseparable from their origin and outcome (books acting as starting points of production/consumption for instance), where is ground-zero fact? Is it through philosophy, science, or could it be in aesthetic rhetoric?

If every object of human knowledge is necessarily altered by the very act of coming to know it, or bringing it before the human mind, then it looks as though the attempt to uncover 'ground-zero' fact is doomed to failure. There seems to be no way, in principle, that you could factor out the contribution made by the object as it is in itself, from the contribution made by human knowledge, perception, concepts, or language.

If the argument I have just given is correct, then there can be no exceptions, not for philosophy, not for science. I'm not sure what you mean by 'aesthetic rhetoric'. As Hegel shows at the beginning of the Phenomenology of Mind, if you try to describe things on a purely aesthetic level, just as they appear prior to conceptualization, you end up babbling.

Is this a worry? Here is what I say in Chapter One of my book Naive Metaphysics, where I compare the knowledge gained through pursuing a philosophical investigation with

...the neurosis which disappears in the course of its being brought to light, the particle which moves when we try physically to observe its position. Yet still, neuroses are uncovered, the positions of particles are observed. In each case, the relation between knower and known reveals an internal complexity, a special dynamic which each form of knowledge works through in its own way. The illusion to be got rid of is that this is somehow second best, the thought that ideally what one would like to have before one's mind is the object itself, rather than the mere knowledge of that object.

However, the implication of your question is that if there is no 'ground zero' fact to be uncovered, then we cannot legitimately speak of truth being 'out there'. The alternative is to say that the truth is 'in us'. Truth is something we make or construct, rather than discover or reveal. In other words, idealism is the inevitable conclusion.

As one of the philosophers who are prepared to take arguments for an idealist or 'anti-realist' conclusion seriously, I think that you have tried to buy an idealist conclusion too cheaply!

Suppose we start off by assuming the existence of facts 'out there', in your sense — something like Kantian 'things in themselves', but without the implication that these things exist outside space or time. Knowledge, you say, always entails bringing these objects in relation to our cognitive faculties. So what we know are only things as they are 'for us', not things as they are 'in themselves'. But note that this conclusion is about knowledge, not about truth. The argument as it stands is not a reductio ad absurdum of the realist belief in facts 'out there', but simply a straight deduction leading to an empty metaphysical scepticism, which leaves all of human knowledge exactly where it was before.

Geoffrey Klempner

back

John asked:

I'm trying to track down any previous exponents of the following notion: One way to make sense of the idea of survival after death is to posit that what survives is the "idea" or "thought" of the person, as retained in the "mind of God." For this to work, you also have to posit that a person's identity (understood as the seat of the conscious "I") is a non-physical thing that could exist as a thought. The analogy might be to a piece of music: clearly, in some sense the Moonlight Sonata exists as an idea, independent of any instantiation in performance or recording. So even if all physical/aural instances of it are wiped out, as long as I or you remember it, it exists. Similarly, as long as God remembers us, we exist after our physical "performance" is over. This notion has some problems, but I'm wondering if you've run across it before.

What you describe sounds very like the Pythagorean doctrine of reincarnation. In his dialogue Phaedo, Plato refers to a theory — which sounds very much like a theory that would have been held by the Pythagoreans — according to which the soul is an 'attunement', like the tuning of a lyre. In the dialogue, the theory is put forward by Socrates' friends Simmias and Cebes. Socrates rejects the theory, as an argument for immortality, on the grounds that if you destroy a lyre you destroy the attunement which existed in that particular instrument. But clearly what the Pythagoreans had in mind was that the same tuning can be instantiated in different lyres. The difference between a human personality and a lyre (or guitar) tuning is that human personalities are, as a matter of fact, unique.

It follows from this theory that the Socrates' special attunement could come to be realized in another living human body. It is only an accidental fact that we never come across the same human 'tuning' twice.

There is a noticeable gap in this theory: Where does the Socrates tuning exist after the death of Socrates' body and before it is realized in another body? I suppose this is where you would say the 'mind of God' comes in. The Pythagoreans, who first discovered the relation between number and harmony, had an ultra-realist view of numbers. So they could have been happy to defend the view that in between bodily realizations, the Socrates attunement exists as a real numerical value, with causal powers sufficient to enable it to impress itself on a new body, without requiring anything to exist in.

As an interpretation of the Pythagorean doctrine of reincarnation, this is to a certain extent speculative. Apart from alleged reports of cases of reincarnation, such as that of Pythagoras himself, the only Pythagorean-sounding argument to be found in the existing texts is the one which Plato rubbishes in the Phaedo.

At this point, an acute reader may have noticed a remarkable coincidence with ideas from AI research. If there is such a thing as a Geoffrey Klempner program, as some AI theorists believe, then, in principle, when my body grows old, my program could be uploaded from my brain onto a computer disc and downloaded into a new, young body, perhaps cloned from one of my own cells. What a great idea! As Daniel Dennett notes in his book Consciousness Explained it promises a much better chance of immortality than having your head chopped off after your death and put into a freezer in the hope that future medical science might be able to get your brain working again. (In the USA, there is — or was — a company that does this for a fee.)

So there is a possibility that I might survive the death of my present body. But there's a catch, the same catch as there is with the Pythagorean theory, and with your theory of a person as a 'thought' in the mind of God. Being quite adept at problem solving, the GK program is too good to wasted on just one body. So, without my knowledge, pirated copies are downloaded into several more bodies. Then one day we meet up. — I'll leave you to write the rest of this scenario. (If you're stuck for ideas, check out my science fiction story The Insurance Policy.)

Geoffrey Klempner

back

Laurence asked:

What is the practical effect of Nietzsche's theory of eternal recurrence? Whether life is a one time performance in eternity or is re-run (that we don't know is a re-run) until the heat death of the universe doesn't seem to make any difference to the conclusion that you had better live life large this time around, because there isn't any rewind button. ER would thus appear to be a metaphysical exclamation point rather than have any independent relevance.

This makes an interesting counterpoint to the previous question. First, what is Nietzsche's theory of eternal recurrence? The theory was originally held by the Stoics. According to Nietzsche, given a deterministic universe of matter in motion, and sufficient time, sooner or later the material particles will fall into a pattern that has existed before. From that point onwards, determinism entails that the very same events will occur that occurred the previous time around until that configuration is reached again, and so on to infinity.

I seem to recall that there is a fatal flaw in that argument due to Nietzsche's failing to take into account irrational numbers. But I've forgotten how it goes. Perhaps you can work it out for yourself. The important thing is that, even if it is not necessarily the case that the same pattern will ever recur, if it does recur then it will indeed, as Nietzsche says, recur an infinite number of times. — That rules out a 'heat death' for the universe, incidentally.

For Nietzsche, the importance of the idea of eternal recurrence lies in the thought that it will indeed be I that returns. This tapping of the keys to produce these worlds will occur an infinite number of times, each time accompanied by these same thoughts. Would this thought crush you, asks Nietzsche? Or would you affirm it with a joyous, Yes! — I just don't know.

The problem is I don't see any interesting sense in which the GK who will exist the next time around, or the GK who existed the previous time around, or all the other past and future GK's can be me. As you say, it is not as if we have been given a 're-wind' button. When you re-wind a music tape to listen to it again, you are aware of hearing it as a re-run because you remember hearing it before. By hypothesis, none of the GK's have the slightest awareness of the experiences of any of the previous GK's. If they did, they would not be realizing the same configuration that existed before.

The philosopher Timothy Sprigge, who gives an appreciative account of Nietzsche's theory in his excellent Penguin paperback Theories of Existence, takes the opposite view from the one I have just expressed. It will be me who will come back into existence when the universe turns around. However, Sprigge doesn't make any attempt to provide an argument for that conclusion, and I seriously doubt whether one could be given.

There is just a tiny seed of doubt in my mind that I may simply have failed to make the imaginative leap required to appreciate the point Nietzsche is making. Really digging down, my only thought is this: In certain moods, I do find something terrifying about the realization that every moment of time as it passes by is gone, never to return. If Nietzsche felt that way too (I don't know of any place where he owns up to this chronophobic thought) then I can see how the notion of eternal recurrence might seem a kind of metaphysical comfort.

Geoffrey Klempner

back

Helen asked:

Buddhists (and other religions as well) say that the only real truth about reality derives from a meditative experience that passes beyond concepts, language, and rationality whereas the whole project of philosophy is of necessity bounded by language. Which is right path, meditation or reason?

The idea of "meditative experience that passes beyond concepts, language, and rationality" is itself is a statement. The modernist French philosopher Maurice Blanchot entitled one of his books, le pas au-dela, translated as "the step beyond" but it is a play on words, because it also means "the not beyond". The step beyond is always the not beyond. The step beyond language is always not beyond language. There is no such thing as experience without language. The meditation posture, the lotus position, like all bodily gestures, belongs to language which we understand. The very 'lotus' metaphor, for instance, of a beautiful flower which grows out of the mud, is symbolic, as is the posture itself. The silence and apatheia of meditation is deeply resonant, but only because it belongs to the realm of language.

"Beyond rationality" is a statement that appeals to reason, so that you think, 'yes, I want that! I must meditate.' Reasonable enough. Buddhism, like any religion, has its own rationale; Buddhism especially. Meditation is not opposed to reason except if you hold (and then you would have to hold tight) a narrow idea of both. If we take reason in the narrow sense of the rationalism of logical positivists or strict empiricists, then almost the whole of the living world is out of gear with reason. But if we take 'reason' in the proper, living sense that our language is always already reasonable, which grammar has to be to make sense, and we have to be to understand each other, then the whole of religion belongs to that sphere. The opposition of meditation to reason is a false dichotomy. I imagine that neither can properly do without the other.

Matthew Del Nevo
www.sicetnon.com

back

Jorge asked:

God does exist?

In Western culture there are two ideas of God: that derived from Judeo-Christian thinking and that derived from Greek philosophy.

The view derived from Greek philosophy has to be some kind of metaphysical postulate, like Aristotle's Unmoved Mover, or Plato's Good; Spinoza's substance or Hegel's Geist. Karl Jaspers has called the so-called God of the philosophers, "the unconditional imperative". It is not an object of knowledge. We infer the unconditional imperative from our natural world with our natural reason. Historically, the greatest philosophers have been those whose thought has brought to light, by way of pure presupposition, an idea of the unconditional imperative which has hitherto been unthought.

The Judeo-Christian God is a God who is known through Revelation. This is the God of Abraham, Isaac and Jacob. This is the God we tend to wonder about the existence of, not the God of the philosophers.

I think of it like this: everyone, whoever they are, kneels before something, whether it be money, fame, knowledge, power, another person. I mean kneel in a metaphorical sense. Our God is that before which we kneel, because kneeling means offering ourselves, veneration of the other, worship. What we worship is God. So in a sense, there is always a God before each one of us, whether we like it or not.

In this context we might like to reconsider the ontological argument put forward by Augustine in the fifth century, but reformulated in scholastic manner by Anselm in the eleventh, then refuted by Kant but revived by Hegel. Anselm said, God is that than which nothing greater can be conceived. If we can conceive something greater, then it is not God we are thinking of, but an idol of our own imagining. This is a way of saying that God is neither conceivable nor inconceivable. Not conceivable because we can always think of something greater eg. 'greater than the greatest'. Not inconceivable because the idea of that than which nothing greater can be conceived always lies before our mind's eye like a horizon line beyond which we can't see. But if we simply say 'there is nothing beyond that horizon' we can think of something greater: a horizon beyond which there is something (even though inconceivable as to what). If you can think this, you are not thinking of God, but your thought is directed toward an ultimate horizon beyond which something is.

— Given this, hopefully you will be able to draw your own conclusions.

Matthew Del Nevo
www.sicetnon.com

back

Emme asked:

What is Will according to Schopenhauer? Did his notion of Will change as time passed?

You probably read my answer about Schopenhauer's will given to Wang. Schopenhauer distinguishes between a realm of manifestation evident to the senses and that realm which is the source of manifestation, out of which phenomena show themselves as what they are. The realm of all things is like a circumference which is drawn around the will which is the primal source of all things; the Being of beings. "The world is only the mirror of this willing." (World as Will and Representation IV.63). The world empiricists call 'real' Schopenhauer calls 'representation'. The real world is the essence of representation, which is the will. "The present is the only place where the will manifests itself" (WWR IV.54 His underscore). So you see, the will is a theory of the nature of being and time.

Did his notion of will change as time passed? Not really. Schopenhauer lived between the years 1788 and 1860. The World as Will and Representation (Der Welt als Wille und Vorstellung) his masterpiece, came out in 1818. It is an intuitive theory and Schopenhauer expects his reader be intuitive as well. Everything he wrote afterward is based on it and backs up what he thought then. The main thing he wrote before, his doctoral thesis, was entitled, On the Fourfold Root of Sufficient Reason. This rigorous work is very much a precursor to The World as Will and Representation, working out, as it does, some of the conceptual underpinning which gives rise, for Schopenhauer, to the 'practicality' of the distinction between what he would come to see as will and representation.

Matthew Del Nevo
www.sicetnon.com

back

Russel asked:

Whenever I have been taught any of the Hume's theories I find myself stuck at the same point every time. If I have understood him correctly then he takes the idea of empiricism to an illogical conclusion. i.e. that nothing can be established or proved outside of empirical experience. If this is true then how can he appeal to his intellectual faculties in order to justify this claim? It seems he relies on the very thing he wants to discredit in order to arrive at his conclusion.

It would have been perfectly consistent for Hume to argue that although human beings possess an autonomous faculty of reason, it cannot be used to prove any conclusion beyond the immediate facts of subjective experience. Induction from past experience to future experiences cannot be rationally justified. Nor can we justify belief in a necessary connection between an effect and its cause, or in an external world of objects distinct from our perceptions.

In this scene of desolation, logic and reason remain intact. All human knowledge, on this picture, consists in knowledge of our immediate experiences, together with knowledge of the relations between 'ideas', Reason is discredited only when it seeks to establish conclusions that transcend its (very narrow) limits.

Yet it is clear that Hume wanted to go further, reducing reason and logic to the psychology of the 'association of ideas'. On this 'psychologistic' theory, if we feel ourselves to be rationally persuaded of the truth of Hume's conclusions, all that is really going on, according to Hume, is a kind of brainwashing. And that seems fatally to undermine his case. If someone says, 'I am not trying to persuade you, I am only trying to brainwash you,' Then we are free to walk away.

I think Hume can be defended against this objection. We can summarize the radical Humean position as the claim that reason is impotent. Let us assume that in order to rationally prove that reason is impotent it is necessary to assume that reason is not impotent. Then Hume can say:

1. Either reason is impotent or not.

2. If you accept that reason is impotent, then I'm home.

3. If you think that reason is not impotent then the argument in my book rationally shows that reason is impotent. Then I'm home.

4. So whether or not you accept my claim straight off, reason is impotent.

— In other words, you can't accuse Hume of relying on the assumption that reason is not impotent in order to establish his radical empiricist conclusions. That assumption is merely the discarded premiss of an argument that proceeds by reductio ad absurdum.

Geoffrey Klempner

back

John asked:

How can humankind in general advance when so many get drunk, shoot guns and act like idiots?

Your question implies that they can't. The philosophy that things are advancing (or if they are not, they should be) is questionable. The First Noble Truth of Buddhism that all life is suffering, or the teaching of Christianity about Original Sin, suggest that the idea of advance is either an illusion (Buddhism) or a natural impossibility (Christianity). The idea of advance became dominant in the Nineteenth Century when the Western world began to get very rich. Today advance it is a popular concept in the media. If inflation is down and the economy is strong we imagine we are advancing. But the implication of your question is probably right, we are not really advancing in any moral sense. The great Nineteenth Century historian Ranke said, "Every generation is equidistant from eternity." Your question seems to imply as much.

Matthew Del Nevo
www.sicetnon.com

back

Jose asked:

Why do philosophers always begin with etymology when they define a word?

Simple. The origin of word tells us a great deal about the concepts being invoked when the word is used.

For example, if we wish to analyse 'democracy', but do not examine the etymology of the word, we would conclude that democracy was, by definition representative; that it involves things like parliaments, presidents or prime ministers and so on; and things of that ilk. Here we are being descriptive. We are taking the meaning of the concept to be that which it is commonly applied to. If we do this, mistakes, such as claiming that the old DDR (East Germany) was democratic because it had the word 'democratic' in its name, occur.

If we examine the etymology of 'democracy' we see that it comes from the French democratie via late Latin from Greek democratia, from demos 'the people' plus kratia 'power, rule'.

We therefore know far more about the concept than had we not examined the etymology. We may even be able to assess whether something is democratic or not. Without the etymology, all that was called democratic was democratic, which is a clear misunderstanding.

Finally, etymology is interesting.

Steven Bullock
University of Stirling

back

Mohammed asked:

I know that deduction is a kind of inference in which the truth of premises determine/prove the truth of the conclusion, induction is a kind of inference in which the truth of premises just supports the truth of the conclusion.

But I've read some other texts which have confused me, expressing that the direction of inquiry from knowledge of a universal to knowledge of its particulars is 'deduction', and that from knowledge of particulars to knowledge of their universal is 'induction'. If the latter definitions are wrong then tell me what labels/terms should be given to those types of inquiry.

Here are some key characteristics of deductive and inductive arguments that should help clear up your confusion:

Deductive:
  1. There is no possible situation where all the premises are true and the conclusion false. In other words if the premises are true the conclusion must be true.
  2. All the information in the conclusion is already contained in the premises.
  3. The inference moves from a universal to a particular claim.

Inductive:

  1. If the premises are true, the conclusion is probably true.
  2. the information in the conclusion exceeds that given in the premises.
  3. The inference moves from a particular claim to a universal or law-like conclusion.

So the reason why the inference in deductive arguments involving universal propositions is from universal to particulars rather than the other way round is because we have all the information about what the conclusion will contain already in the premises. Let me give an example:

Premise 1: All monkeys drink Guinness.
Premise 2: Harry is a monkey.
Conclusion: Harry drinks Guinness.

Premise 1 is a law like, universal statement. (It does not matter that Premise 1 is actually false, since we are concerned with any possible situation. It could be that in an alternative universe monkeys do drink Guinness.) Premise 2 tells us about a monkey, i.e. Harry, so we already know, at least implicitly, that Harry drinks Guinness. The inference to the conclusion makes that knowledge explicit.

Moving on, induction has caused heaps of people heaps of trouble, so we will keep it simple. In inductive arguments we start with some claim (usually based on experience) and infer some general conclusion. For example:

Premise: Every time I have been fishing in the past I have caught a big fish.
Conclusion: Every time I go fishing I will always catch a big fish.

The conclusion is a general claim but clearly it is at best only probable since the day may come when I do not catch a big fish. So in induction the premise does not provide any guarantee of the truth of the conclusion, since the conclusion is meant to go beyond the information we have in the premise.

Brian Tee
Dept of Philosophy
University of Sheffield.

back

Duane asked:

I think this is from Descartes:

a) For any two things, if they are identical, then all statements describing one of those things must describe the other.
b) I can picture existing without my body.
c) I can not picture existing without my soul.
Ergo,
d) My soul and my body are not identical (i.e. Cartesian dualism obtains)

Hmm...I always like these simple arguments, please comment. Thanks. (I wonder if I could put that argument into some sort of symbolic logic. . .)

Yes it is from Descartes and I like these simple arguments also. The trouble with this one is first trying to decide how Descartes is arguing — there are many different interpretations.

The first and most common interpretation is that Descartes is appealing to what is now called Leibniz's Law. "If we can say something true about A, which is not true about B, then A and B are not the same thing." If this is how Descartes is arguing then it is clearly invalid since there are many situations where Leibniz's law breaks down. For example, I may believe that Elvis is a bar tender, but believe that the man I am now looking at is not a bar tender and therefore that this man is not Elvis. Nevertheless, it may be that the man is Elvis. This objection applies to the argument you presented in your question so that simple argument is simple to invalidate.

A second interpretation is by a fellow called Hooker (Hooker, M. 'Descartes denial of mind-body identity' reprinted in Descartes: Critical and Interpretative Essays John Hopkins UP 1978). This is definitely not simple, but I think it is valid:

1. I can conceive of myself existing and no bodies existing.
2. For all propositions P, if P is conceivable then P is possible.
3. Therefore, it is possible that I exist and no bodies exist.
4. For all x, if x is a body, then x is essentially a body.
5. Therefore, if I am a body, then I am essentially a body.
6. If I am essentially a body, it is not possible that I exist and that no bodies exist.
— Contradiction!
7. Therefore, I am not essentially a body.
8. Therefore (from 5) I am not a body.

If this argument is valid we must question the premises, if we want to argue against it. I think premise 1. is the most questionable. What would it be like to exist without a body? Even if we could imagine what it would be like, is disembodiment a real possibility?

Brian Tee
Dept of Philosophy
University of Sheffield.

back

Sheila asked:

What do you feel is St. Augustine's position on original sin?

Also, what do you think is the source of all knowledge according to John Locke and how do you think these elements function in creating our view of the world?

Augustine literally believed St. Paul where he says that sin "entered the world by one person and spread to everyone" (Rom. 5:12). In other words, human existence is fundamentally disordered at the source. His Commentary on Genesis (De Genesi ad litteram) makes fascinating reading in this regard. The 'original sin' (peccatum originale) of Adam was pride, the false self-love that makes the self its own end. In an early work, On Free Will (De Libero Arbitrio) Augustine writes, "The human will sins when it turns aside from the changeless and universal Good, and turns towards its own private good, or to goods remote from, or beneath itself. It turns to a private good when it wills to be in its own power; to a good outside its proper range, when it seeks the knowledge of what belongs to others or concerns not itself; to an inferior good when it loves bodily pleasure. Thus, a person, given over to pride, curiosity or wantonness, finds himself in another life, which in comparison with the higher life, is death ... What is evil is the will's aversion from the changeless Good and its conversion to the goods that are changing; and this conversion, being voluntary and not compelled, is followed by the fit and just punishment of misery." (De Lib. Arb. II. 53.) In his Retractions written at the end of his life, in which Augustine went back through his writings and made retractions of things he wished he had not said, we find him considering this definition of sin as 'natural' and willful and sticking by it.

The source of all knowledge for John Locke was experience, conceived as either external (sense data) or 'inner' (reflection upon materially produced data). In either case sensation accompanies experience and reflection works on sensation in order to understand it. Locke is very mechanistic in his thinking. Because Locke calls the possibility of the self knowledge into question by his presumption that the so-called 'self' is a blank slate (tabula rasa), a naturally sceptical and materialistic world view is the outcome. Historically, Locke's philosophy gave metaphysical justification for the 'enlightened worldliness' of his time. However, his questioning of the idea of knowledge — that it is abstracted from experience and then fabricated by means generalization and association — paved the way for Humean scepticism, in which knowledge is reduced to subjective impressions. But then along came Kant...

Matthew Del Nevo
www.sicetnon.com


Second opinion:

St Augustine's position on original sin

The doctrine of original sin originates not with the story of the fall of Adam and Eve found in Genesis 3, for this text does not say that we are subject to their punishment, but with the interpretation found in the Epistle of Paul to the Romans. It is from here that Augustine formulates and finds scriptural support for his position on original sin.

Augustine's conception of original sin is morally questionable for two reasons:

1. The old problem of reconciling God's nature as an all-loving being with the fact that when Adam sins God punishes him, and the whole of humanity to follow with death.

Further, there is the question of whether God has any just claim to be the author of morality. If all that constitutes morality is the will of God and to sin is to go against the will of God then this is not morality at all but prudence, since the only reason we do as God says is not because we see it as right or good, but because we do not want to die. And if morality is independent of God's will then maybe Adam was right to go against God's will and so what he did was not even sinful.

2. Leaving that problem aside, there is the general question of whether sin or moral guilt can be transferred from one person to another by any causal mechanism let alone the biological "sin gene". Some Christians believe that such a transfer is possible and was accomplished in the trials and tribulations of Jesus Christ. The idea is that Jesus died as payment to God for our sin. (For an entertaining interpretation of this theology see Nietzsche's The Anti-Christ.)

Richard Swinburne, a Christian philosopher criticises Augustine's ideas on original sin along these lines. Swinburne argues that no one can be guilty for the sins of any other person unless he had an obligation to deter that person and failed to fulfil that obligation. Since no one alive today (or even then) could have stopped Adam and Eve from sinning we cannot be guilty for their sins. In fact the one person who could be said to have an obligation to Adam and Eve was God himself. If he could have stopped them and didn't then God is responsible for the first sin. Of course that is a big 'If', raising questions about free will and God's omnipotence.

So it seems that we have good reasons to reject Augustine's conception of original sin.

Locke on the source of all knowledge

At the beginning of Book Four of his Essay on Human Understanding Locke defines knowledge as "Nothing but the perception of the connection and agreement or disagreement and repugnancy of any of our ideas". So knowledge derives from reasoning about our ideas, and ideas according to Locke originate in experience.

Locke draws certain conclusions from this definition about the extent of knowledge. One is that, "We can have knowledge no further than we have ideas". Our knowledge cannot go beyond the ideas or concepts that we have, and the ideas that we have are bounded by experience. (Just a point of clarification. While Locke holds that all ideas derive from experience, he does not believe that all our claims to know are justified by appeal to experience. Instead, knowledge is a relation between us and the perceptions we see between our ideas.)

What does this mean for our view of the world? Locke thinks that knowledge is only possible in such areas as mathematics, logic and ethics. In other areas such as metaphysics and science we can only have degrees of probable belief. Why is this? Given his definition of knowledge, Locke thinks that we are unable to perceive any connection between the constituent ideas of a scientific hypothesis or an empirical observation. All we do perceive is the conjunction of ideas without any necessary or intrinsic connection. This is because in the case of material objects (the external world) we cannot know what their real essences are, because these real essences can never be objects of human perception. Locke does not think, however, that this lack of knowledge should lead us to scepticism. Instead, he argues that knowledge is not required in many instances. Probable belief is satisfactory for our purposes in acting in the world.

So for Locke knowledge is severely limited for us, restricted to the scope of necessary truths. We can nevertheless have degrees of certainty about aspects of the world. One problem for Locke's account is that while we may be able to perceive connections between ideas, we have no guarantee that anything corresponds in the world to our ideas. In other words, the problem for Locke is the lack of any connection between our ideas and the existence of that which our idea is an idea of. Without any such reassurance, how can we make any claims about the world? All we know is the ideas we have and the connections between them. Our world then becomes confined to the realm of necessary truths, logical relations.

While Locke thought that there was an intermediate stage between certain knowledge and scepticism, the fact that all one can know is the inside of one's own head, and we have no way of knowing whether what we think corresponds to anything real outside us, means that we can only have a limited conception of the world.

Brian Tee
Dept of Philosophy
University of Sheffield.

back

Lev asked:

In how many ways does mind differ from matter?

It all depends on what theory you find more attractive. If you are a materialist then there is no difference, the mind just ismatter. If you are an idealist, them matter is just a collection of ideas in the mind. If you are a dualist then there are many differences. Descartes thought that the main difference between mind and matter is that matter is a substance that is extended in space and time, this is the essence of matter. The essence of mind, on the other hand, is to think or to be conscious.

If you looked at a philosophy book, you would probably see a list like this one:

MATTER
extended
locatable in space
publicly observable
causally determined
objective
cannot be "about" anything

MIND
not extended
not locatable in space
private to one person only
possesses free will
subjective
able to represent other things

I do not think that the mind is a thing, a substance, unlike Descartes who did think that, but neither do I think that the mind is equivalent to matter. Instead, I would argue that the main difference between mind and matter lies in what is called the Qualitative aspect of the mental. To have a mind, to be conscious means that there is some way it feels to be having those experience. This qualitative character is subjective, that is, whatever experiences I have belong to me in a way that they do not belong to anyone else, and although others may know what experience I am having, they do not know what it is like for me to have these experiences.

Brian Tee
Dept of Philosophy
University of Sheffield